¿Cambia el momento angular de giro con el marco de referencia?

Supongo que el momento angular de una partícula medido por diferentes observadores inerciales (sin gravedad) es diferente. Al mismo tiempo, dado que el espín cuántico es una propiedad intrínseca,

  1. ¿Es cierto que el momento angular de espín es invariante de Lorentz?
  2. Si es así, ¿cómo se concilia con el hecho de que, al ser un momento angular, debería cambiar?
  3. dado el momento angular total de alguna partícula desconocida, ¿hay alguna forma de dividirlo en espín y otro momento angular?

Estoy pensando que (1) es cierto, pero estoy indeciso sobre 2. y 3. Se agradece cualquier ayuda.

Tal vez para 2., todo lo que la transformación de Lorentz está obligada a hacer es cambiar el momento angular total, pero no necesariamente sus contribuyentes, por lo que el giro en sí podría permanecer constante. En este caso, ¿no sería 3. afirmativo?.

La magnitud del espín es invariante de Lorentz, pero el espín en sí es un cuatro vector (o cuatro tensor antisimétrico de dos índices) que se transforma de acuerdo con las reglas habituales de transformación de Lorentz.
@ G.Smith, ¿cambia el vector de giro 4 independientemente del vector de momento angular total 4?
No solo puede tener una dinámica independiente, sino que puede existir de forma independiente. Puedes tener giro sin ningún momento angular orbital.

Respuestas (1)

El espín de una partícula se define por su momento angular intrínseco y, como tal, se define en el marco de reposo de la partícula. Entonces, su vector de giro cuatro en el marco de descanso debe tomar la forma

S m = ( 0 , S )
Donde el 3-vector S es su conocido giro no relativista.

El cuatro vector se transformará precisamente así, en la representación vectorial del grupo de Lorentz, por lo que en un marco arbitrario tendrá componentes

S m = ( s , S ) ,
donde como la longitud del vector es invariante debemos tener s 2 + S 2 = S 2 . De hecho, podemos demostrar que la componente cero del vector de espín es proporcional a la helicidad de la partícula (una invariante): s = h | pag | dónde pag es el momento (no invariante) de la partícula en cualquier marco en el que se encuentre y h es su helicidad. La helicidad es esencialmente la proyección del giro de la partícula en la dirección de su movimiento, como tal, para las partículas sin masa, el giro coincide con la helicidad.

Podemos dar una forma explícita para el vector de espín siguiendo a Pauli-Lubanski que tiene las propiedades mencionadas anteriormente. Let (su notación)

W m := 1 2 ε m v α β PAG v METRO α β
Con PAG v las componentes de los cuatro impulsos y METRO α β los generadores (valorados en matriz) del grupo de Lorentz (para los vectores, esto es solo el momento angular orbital, pero para la representación de giro 1/2 hay una pieza adicional que involucra el γ matrices). Tenga en cuenta que en el marco de descanso, PAG v = ( t , 0 ) y la antisimetría del tensor Levi-Civita implica que W 0 = 0. podemos obtener una relación más general que esta al notar que en todos los marcos de referencia PAG m W m = 0 por lo que el vector de espín es covariantemente ortogonal a la velocidad de la partícula.

De hecho, el (pseudo-)vector de Pauli-Lubanski es un Casimiro del grupo de Poincaré y se usa ampliamente en la clasificación de representaciones irreducibles del grupo de Lorentz.

Finalmente, OP preguntó sobre las contribuciones orbitales (L) y de espín (S): los generadores de Lorentz se construyen a partir de la suma de un término orbital y un término de espín,

METRO m v = X m pag v X v pag m + S m v
donde el término final depende del modelo (para el campo de Dirac es proporcional a [ γ m , γ v ] ). Los generadores se transforman en el representante de dos tensores del grupo de Lorentz, lo que asegura que el vector de espín se transforme en el representante fundamental, es decir, como un vector de cuatro.

Entonces, su vector de giro cuatro en el marco de descanso debe tomar la forma Sμ=(0,S) Esta lógica no funciona. Si lo aplicamos a otras cantidades, como la fuerza o el campo eléctrico, da un resultado falso.
@lux gracias por la respuesta detallada (explicando conceptos que hasta ahora desconocía). Sin embargo, ¿puede explicar el razonamiento en el contexto de las preguntas específicas que he hecho? Más precisamente, entiendo que la respuesta a 1 es falsa ya que S es un vector 4, pero ¿cómo se transforma cuando se ve desde la perspectiva de ser un momento angular ordinario? seria el total L + S se transforman o se transforman por separado?
El vector de giro se define en el marco de reposo como se indica: ¿qué es el giro sino el momento angular observado cuando su parte orbital desaparece? Consulte en.m.wikipedia.org/wiki/Relativistic_angular_momentum#Four-spin para ver que esta no es mi definición sino la aceptada. Estoy de acuerdo en que no funciona para otras cantidades, pero no sugiero aplicar la misma lógica a la fuerza o los campos eléctricos.
En cuanto a la separación en contribuciones orbitales y de espín, ahora he editado la respuesta.